Đến nội dung

xuanhoan23112002 nội dung

Có 95 mục bởi xuanhoan23112002 (Tìm giới hạn từ 29-03-2020)



Sắp theo                Sắp xếp  

#712421 $2^n-1$ là số nguyên tố

Đã gửi bởi xuanhoan23112002 on 12-07-2018 - 22:34 trong Số học

Bài này dùng phản chứng thôi.




#712416 Đề thi IMO 2018

Đã gửi bởi xuanhoan23112002 on 12-07-2018 - 21:52 trong Thi HSG Quốc gia và Quốc tế

Kết quả chính thức IMO 2018



#711677 $f(x)=ax^4+bx+c> 0 \forall x> 0$

Đã gửi bởi xuanhoan23112002 on 27-06-2018 - 16:55 trong Đa thức

Cho $a\neq 0$ và $f(x)=ax^4+bx+c> 0 \forall x> 0$

CMR: $f(x)$ được biểu diễn ở dạng tổng bình phương của 2 tam thức bậc hai.




#710198 AP vuông góc với IJ

Đã gửi bởi xuanhoan23112002 on 07-06-2018 - 14:17 trong Hình học

Cho tam giác ABC với AC > AB. Các đường cao BB, CCcủa tam giác cắt nhau tại H. Gọi M, N lần lượt là trung điểm của BC', CB'. MH cắt đường tròn ngoại tiếp tam giác CHB' tại I; N

H cắt đường tròn ngoại tiếp tam giác BHC' tại J. Giả sử P là trung điểm cạnh BC. Chứng minh: AP vuông góc với IJ




#710187 Tâm đường tròn ngoại tiếp tứ giác ABCD nằm trên SI

Đã gửi bởi xuanhoan23112002 on 07-06-2018 - 11:51 trong Hình học

Cho tứ giác ABCD ngoại tiếp đường tròn tâm I. Giả sử bên trong tứ giác ta vẽ được 4 đường tròn bằng nhau và cùng đi qua 1 điểm S, và mỗi đường tròn tiếp xúc với 2 cạnh liên tiếp của tứ giác đó. Chứng minh tứ giác ABCD nội tiếp 1 đường tròn và tâm đường tròn đó nằm trên SI.




#710186 Đề thi tuyển sinh vào lớp 10 THPT chuyên tỉnh Vĩnh Phúc năm học 2018-2019

Đã gửi bởi xuanhoan23112002 on 07-06-2018 - 11:41 trong Tài liệu - Đề thi

Câu 3 (phỏng theo lời giải của thầy Võ Quốc Bá Cẩn) 

Ta có: $\frac{a^2}{a^2+ab+b^2}+\frac{c^2}{c(a+b+c)}\geq \frac{(a+c)^2}{a^2+b^2+c^2+ab+bc+ca}$ (bất đẳng thức Schwarz)

Làm tương tự với 3 phân thức còn lại ta có:

$\frac{a^2}{a^2+ab+b^2}+\frac{b^2}{b^2+bc+c^2}+\frac{c^2}{c^2+ca+a^2}\geq 1$

Đẳng thức xảy ra $\Leftrightarrow a=b=c> 0$

Vậy ta có điều phải chứng minh.




#710109 Đề thi tuyển sinh vào lớp 10 THPT tỉnh Bắc Giang năm học 2018-2019

Đã gửi bởi xuanhoan23112002 on 06-06-2018 - 15:07 trong Tài liệu - Đề thi

Câu 5: Ta có:

$P=\frac{81x^2+18225x+1}{9x}-\frac{6\sqrt{x}+8}{x+1}\geq \frac{18x}{9x}-\frac{9x+9}{x+1}+2025= 2018$ (bất đẳng thức Cauchy)

Đẳng thức xảy ra $\Leftrightarrow x=\frac{1}{9}> 0$

Vậy $MinP=2018\Leftrightarrow x=\frac{1}{9}$




#710107 Đề thi tuyển sinh vào lớp 10 chuyên toán tỉnh Kiên Giang

Đã gửi bởi xuanhoan23112002 on 06-06-2018 - 14:53 trong Tài liệu - Đề thi

Bài 7: Bài bất đẳng thức có vẻ dễ nhỉ

Ta có:

$\frac{x^2}{y+z}+\frac{y^2}{z+x}+\frac{z^2}{x+y}\geq \frac{(x+y+z)^2}{2(x+y+z)}= \frac{x+y+z}{2}=1$ ( bất đẳng thức Schwarz)

Đẳng thức xảy ra $\Leftrightarrow x=y=z=\frac{2}{3}$

Vậy ta có điều phải chứng minh.




#710098 [TOPIC] ÔN THI BẤT ĐẲNG THỨC $\boxed{\text{THPT CHUYÊN}}$...

Đã gửi bởi xuanhoan23112002 on 06-06-2018 - 12:08 trong Tài liệu - Đề thi

Bài 148: Cho x, y, z là các số thực dương và $x+y+1=z$. Tìm giá trị lớn nhất của biểu thức sau:

$P=\frac{x^3y^3}{(x+yz)(y+zx)(z+xy)}$




#710076 Đề thi tuyển sinh vào chuyên Lê Quý Đôn tỉnh Quảng Trị năm 2018-2019

Đã gửi bởi xuanhoan23112002 on 06-06-2018 - 07:55 trong Tài liệu - Đề thi

Câu 6:

Từ giả thiết kết hợp với công thức khai triển bậc 4: $(a+b)^4=a^4+4a^3b+6a^2b^2+4ab^3+b^4$ ta có:

$Q=(x+3-x)^4-4x(3-x)(x^2+(3-x)^2)=81-2(9-x^2-(3-x)^2)(x^2+(3-x)^2)=81+2(x^2+(3-x)^2)^2-18(x^2+(3-x)^2)=2(x^2+(3-x)^2-5)^2+2(x^2+(3-x)^2)+31\geq 10+31=41$

Đẳng thức xảy ra  $$\Leftrightarrow x^2+(3-x)^2=5\Leftrightarrow 2x^2-6x+4=0\Leftrightarrow x=1, x=2$$

Vậy $Min Q=41\Leftrightarrow x=1, x=2$




#710050 [TOPIC] ÔN THI BẤT ĐẲNG THỨC $\boxed{\text{THPT CHUYÊN}}$...

Đã gửi bởi xuanhoan23112002 on 05-06-2018 - 20:01 trong Tài liệu - Đề thi

Bài 145: Cho a, b, c là các số thực dương thỏa mãn: $abc=1$. Chứng minh rằng:

1. $\frac{1}{(1+a)^2}+\frac{1}{(1+b)^2}+\frac{1}{(1+c)^2}+\frac{1}{a+b+c+1}\geq 1$

2. $\frac{a+3}{(a+1)^2}+\frac{b+3}{(b+1)^2}+\frac{c+3}{(c+1)^2}\geq 3$




#710048 Đề thi tuyển sinh vào lớp 10 THPT chuyên tỉnh Vĩnh Phúc năm học 2018-2019

Đã gửi bởi xuanhoan23112002 on 05-06-2018 - 19:45 trong Tài liệu - Đề thi

Câu 5:

Từ giả thiết ta có: $c=a+b-\sqrt{ab}$

$P=\frac{c^2}{ab}+\frac{c^2}{a^2+b^2}+\frac{\sqrt{ab}}{a+b}$

$P\geq c^2(\frac{1}{2ab}+\frac{1}{2ab}+\frac{1}{a^2+b^2})+\frac{\sqrt{ab}}{a+b}\geq \frac{9c^2}{(a+b)^2+2ab}+\frac{\sqrt{ab}}{a+b} \geq \frac{6(a+b-\sqrt{ab})^2}{(a+b)^2}+\frac{\sqrt{ab}}{a+b}=6-\frac{11\sqrt{ab}}{a+b}+\frac{6ab}{(a+b)^2}=6(\frac{\sqrt{ab}}{a+b}-\frac{1}{2})^2-\frac{5\sqrt{ab}}{a+b}+\frac{9}{2}$ $\geq -\frac{5}{2}+\frac{9}{2}=2$ ( theo các BĐT AM-GM và Schwarz)

 Đẳng thức xảy ra $\Leftrightarrow a=b=c>0$

Vậy $MinP=2\Leftrightarrow a=b=c>0$




#709905 $a^2 + b^2 + c^2 + abc = 4$

Đã gửi bởi xuanhoan23112002 on 04-06-2018 - 12:38 trong Bất đẳng thức và cực trị

Tìm max: http://diendantoanho...-định-năm-2018/ (chỉ việc thay mỗi số 2 thành số 1 thôi a trình bày đầy đủ rồi)

Tìm min:

Nếu cả 3 số a, b, c đều > 2 hiển nhiên suy ra điều vô lí

Do đó ta giả sử: $c\leq 2$ nên $abc\leq 2ab$

$\Rightarrow 4=a^2+b^2+c^2+2abc\leq a^2+b^2+c^2+2ab=(a+b)^2+c^2\leq (a+b+c)^2$

$\Rightarrow a+b+c\geq 2$




#709874 $a^2 + b^2 + c^2 + abc = 4$

Đã gửi bởi xuanhoan23112002 on 04-06-2018 - 07:45 trong Bất đẳng thức và cực trị

Bạn có thể tham khảo tại đây: http://diendantoanho...2b2c2abc-geq-4/

Đáp án: $Min P=2\Leftrightarrow (a,b,c)=(2,0,0)$ và các hoán vị của nó

             $Max P=3\Leftrightarrow (a,b,c)=(1,1,1)$




#709817 Đề thi lớp 10 môn Toán vào Trường THPT Chuyên Lam Sơn

Đã gửi bởi xuanhoan23112002 on 03-06-2018 - 10:57 trong Tài liệu - Đề thi

Câu 5: Bất đẳng thức đã cho tương đương với:

$a^2b+b^2c+c^2a+ab^2+bc^2+ca^2> a^3+b^3+c^3+2abc$

$\Leftrightarrow a(b-c)^2+b^2(c+a-b)+c^2(a+b-c)> 0$ (luôn đúng đó a, b, c là độ dài 3 cạnh của 1 tam giác)




#709807 Bất đẳng thức

Đã gửi bởi xuanhoan23112002 on 03-06-2018 - 07:54 trong Bất đẳng thức - Cực trị

Cho a, b, c là các số thực không âm. Chứng minh rằng:

$3(a+b+c)\geq 2(\sqrt{a^2+bc}+\sqrt{b^2+ca}+\sqrt{c^2+ab})$




#709806 Tìm GTLN của $2x^2-3xy-2y^2$

Đã gửi bởi xuanhoan23112002 on 03-06-2018 - 07:30 trong Bất đẳng thức và cực trị

Ta có:

$A-36a=(2-25a)x^2-(3+20a)xy-(2-40a)y^2$

Coi phương trình trên là phương trình bậc 2 ẩn x tìm giá trị của a sao cho phương trình có nghiệm kép tức là$\Delta =0$ (Chú ý: Tìm giá trị lớn nhất thì $A-36a$ mang dấu trừ của 1 bình phương đủ nên $2-25a<0, 2-40a<0$)

Từ đó tìm được: $a=\frac{1}{12 }$




#709756 Tìm GTLN của $2x^2-3xy-2y^2$

Đã gửi bởi xuanhoan23112002 on 02-06-2018 - 07:23 trong Bất đẳng thức và cực trị

Đặt $A=2x^2-3xy-2y^2$

$\Leftrightarrow$$A-3=2x^2-3xy-2y^2-\frac{1}{12}(25x^2-20xy+40y^2)$

$\Leftrightarrow$$A-3=-\frac{1}{12}x^2-\frac{4}{3}xy-\frac{16}{3}y^2$

$\Leftrightarrow$$A-3=-\frac{1}{12}(x+8y)^2\leq 0$

$\Leftrightarrow A\leq 3$

Đẳng thức xảy ra $\Leftrightarrow (x, y)=(\frac{4\sqrt{2}}{5}, -\frac{\sqrt{2}}{10})$ hoặc $(x, y)=(-\frac{4\sqrt{2}}{5}, \frac{\sqrt{2}}{10})$

Vậy Max của $2x^2-3xy-2y^2=3$




#709591 [TOPIC] ÔN THI BẤT ĐẲNG THỨC $\boxed{\text{THPT CHUYÊN}}$...

Đã gửi bởi xuanhoan23112002 on 30-05-2018 - 13:38 trong Tài liệu - Đề thi

Bài 140:

Từ giả thiết ta có bất đẳng thức sau: $0< ab+bc+ca\leq \frac{(a+b+c)^2}{3}\leq 1$

Do đó

$\frac{a}{\sqrt{a^2+1}}\leq \frac{a}{\sqrt{a^2+ab+bc+ca}}= \frac{\sqrt{a}.\sqrt{a}}{\sqrt{(a+b)(a+c)}}\leq \frac{1}{2}(\frac{a}{a+b}+\frac{a}{a+c})$ (bất đẳng thức Cauchy)

Chứng minh tương tự như trên ta có:

$P\leq \frac{1}{2}(\frac{a+b}{a+b}+\frac{b+c}{b+c}+\frac{c+a}{c+a})=\frac{3}{2}$

Đẳng thức xảy ra $\Leftrightarrow a=b=c=\frac{\sqrt{3}}{3}$

Vậy $MaxP=\frac{3}{2}$ $\Leftrightarrow a=b=c=\frac{\sqrt{3}}{3}$




#709567 [TOPIC] ÔN THI BẤT ĐẲNG THỨC $\boxed{\text{THPT CHUYÊN}}$...

Đã gửi bởi xuanhoan23112002 on 30-05-2018 - 09:53 trong Tài liệu - Đề thi

Bài 139: Cho a, b, c là các số thực dương thỏa mãn $abc=1$. Tìm giá trị lớn nhất của biểu thức:

$P=\sqrt{3(a^2+b^2+c^2)}(6-\frac{a}{b}-\frac{b}{c}-\frac{c}{a})$




#709566 Số chính phương

Đã gửi bởi xuanhoan23112002 on 30-05-2018 - 09:39 trong Số học

Bài toán này sử dụng phương pháp bước nhảy Viete. Các bài viết khác về bước nhảy Viete trên VMF

http://diendantoanho...ước-nhảy-viete/

Lời giải của bài toán trên bạn có thể tham khảo ở đây: http://math.stackexc...-its-an-integer




#709553 số học

Đã gửi bởi xuanhoan23112002 on 30-05-2018 - 07:32 trong Số học

Từ giả thiết ta thấy ngay a, b, c đều là các số lẻ mà một số chính phương lẻ chia 8 dư 1

Từ nhận xét trên: $a^{30}+b^{4}+c^{2018}\equiv 3$ (mod 8)




#709502 Bất đẳng thức chọn lọc ôn chuyên

Đã gửi bởi xuanhoan23112002 on 29-05-2018 - 15:47 trong Bất đẳng thức và cực trị

Bài 3: Theo giả thiết ta có $0\leq a, b, c\leq 4$ nên

$$(4-a)(4-b)(4-c) \geq 0$$

$\Leftrightarrow 64+4(ab+bc+ca) \geq abc+16(a+b+c)$

$\Leftrightarrow ab+bc+ca\geq 8+\frac{abc}{4}\geq 8$ 

Do đó ta có: $P=(a+b+c)^2-(ab+bc+ca)\leq 36-8=28$

Đẳng thức xảy ra $\Leftrightarrow (a, b, c)=(0, 2, 4)$ và các hoán vị của nó

Vậy $MaxP=28$ $\Leftrightarrow (a, b, c)=(0, 2, 4)$ và các hoán vị của nó




#709479 Bài tập về đa thức

Đã gửi bởi xuanhoan23112002 on 29-05-2018 - 08:53 trong Đa thức

Bài 1: Cho đa thức $f(x)=x^{2018}+\sum a_ix^{i}($a_i\in {-1,1}, $\forall i\in \left \{ 0,1,...,2017 \right \}$$)$ không có nghiệm thực. Tìm số lớn nhất các hệ số = -1 trong f(x)

Bài 2: Tìm đa thức P(x) hệ số thực thỏa mãn:

$(P(x))^{3}-3(P(x))^{2}=P(x^{3})-3P(-x)$, với mọi x là số thực




#709478 [TOPIC] ÔN THI BẤT ĐẲNG THỨC $\boxed{\text{THPT CHUYÊN}}$...

Đã gửi bởi xuanhoan23112002 on 29-05-2018 - 08:41 trong Tài liệu - Đề thi

Bài 138: Với a, b, c là các số thực dương. Tìm giá trị nhỏ nhất của biểu thức:

$P=\frac{2}{a+\sqrt{ab}+\sqrt[3]{abc}}-\frac{3}{\sqrt{a+b+c}}$